Simply: 9+2(3+4)
I need help

Answers

Answer 1

Answer:

23

Step-by-step explanation:

9+2(3+4)

PEMDAS

Parentheses first

9+2(7)

Then multiply

9+14

Then add

23

Answer 2

Answer:

23

Step-by-step explanation:

9+2(3+4)

=9+(2)(7)

=9+14

=23


Related Questions

HELP!!!!!
If anyone knows the answer please tell me as soon as possible PLEASE!!!!

Answers

Answer:

Plotting the points on graph and joining them gives a right angle triangle

Answer:

right angle triangle

Step-by-step explanation:

Slope = (Y1-Y2)/(X1-X2)

The slope of AC is 1/3. The slope of BC is -3. Therefore AC is perpendicular to BC (right angle).

Use special right triangle ratios to find the lengths of the other leg and the hypotenuse

Answers

Answer:

leg = 18

hypotenuse = 18 sqrt(2)

Step-by-step explanation:

We know that sin theta = opp side / hypotenuse

sin 45 = 18 / hyp

hyp sin 45 = 18

hyp = 18 / sin 45

hyp = 18 sqrt(2)

Since this is an isosceles triangle ( the two angles are the same measure), the two legs have to be the same length

leg = 18

the lengths of the other leg and the hypotenuse

is 18 units and 18[tex]\sqrt{2}[/tex]units respectively.

Answer:

Solution given:

Let <C=<B=45°

AB=18 units

BC=?

AC=?

again

By using

By usingspecial right triangle ratios

sin C=opposite/hypotenuse=AB/AC=18/AC

Sin 45=18/AC

AC=18/sin45

AC=hypotenuse=18[tex]\sqrt{2}[/tex]units

again

Tan A=opposite/adjacent=BC/AB=BC/18

Tan45=BC/18

BC=Tan45*18

BC=length of another leg=18 units.

Use the expression, X^2-7
What is the value of the expression above when n=5

Answers

Answer:

18

Step-by-step explanation:

X^2 - 7 =

Since we need to evaluate the expression when X = 5, we replace X with 5.

= 5^2 - 7

Now, according to the correct order of operations, we need to do the exponent first. 5^2 = 5 * 5 = 25

= 25 - 7

Finally, we subtract.

= 18

Answer: 18

ASAP HELP!! PLEASEEEE!!

Answers

Answer:

Step-by-step explanation:

5.1, please this is just a guess from using my head to calculate

HELP PLEASE!

Given that sin A=3/7, cos B=-2/5, and both AA and B are in quadrant II, find cos (A-B). Simplify to a single value and leave it in the form of a rational number.

Answers

First, recall that

cos(A - B) = cos(A) cos(B) + sin(A) sin(B)

so you just need to find cos(A) and sin(B).

Since both A and B end in the second quadrant, you know that

• cos(A) and cos(B) are both negative

• sin(A) and sin(B) are both positive

Then from the Pythagorean identity, you get

cos²(A) + sin²(A) = 1   ==>   cos(A) = -√(1 - sin²(A)) = -2√10/7

cos²(B) + sin²(B) = 1   ==>   sin(B) = +√(1 - cos²(B)) = √21/5

You'll end up with

cos(A - B) = (-2√10/7) (-2/5) + (3/7) (√21/5)

… = (4√10 + 3√21)/35

(which makes the last sentence in the question kind of confusing, because this expression doesn't get much simpler and it's certainly not a rational number)

The value of cos(A - B) is approximately 23/25

Given that A and B are in the second quadrant, we have

sin A = 3/7cos B = -2/5

To find cos(A - B), we have to use trigonometric functions

cos(A - B) = cosAcosB + sinAsinB ...equation(i)

but

cos A

[tex]cos^2A + sin^2A =1 \\cos^2A = 1 - sin^2A\\cos^2A = 1 - (\frac{3}{7})^2 = 1 - \frac{9}{49}= cosA= -\frac{2\sqrt{5} }{7}[/tex]

Having the value of cos A, let's solve for cosB

Cos B

cos B = -2/5

[tex]sin^2B = 1-cos^2B\\sin^2B = 1-(-\frac{2}{5})^2= 1-\frac{4}{25}\\sinB = \sqrt{\frac{21}{25} }=\frac{\sqrt{21} }{5}[/tex]

cos(A-B)

substituting the values if sinA, cosA, sinB, cosB into equation(i) above;

[tex]cos(A-B)=cosAcosB+sinAsinB\\cos(A-B)=(-\frac{2\sqrt{5} }{7})(-\frac{2}{5})+(\frac{3}{7})(\frac{\sqrt{21} }{5})\\cos(A-B)=\frac{3\sqrt{21}+4\sqrt{5} }{35} \\cos(A-B) = 23/35[/tex]

The value of cos(A-B) is given above

Learn more on trigonometric functions here;

https://brainly.com/question/4326804

Which expression is equivalent to 4-2 _ 2-3

Answers

Answer:

16

Step-by-step explanation:

First you calculate the value

(2)÷2^-2

Then you simplify

2^4

=16

write 5 lcms of 100 and 120

Answers

Answer:

The LCM of 100 and 120 is 600.

The LCM of 5 and 120 is 120.

LCM of 5 and 100 is 100.

Step-by-step explanation:

I think this is the answer . If it is not sorry .

The volumes of two similar solids are 512cm3 and 2197cm3. If the smaller solid has a surface are of 960cm2, find the surface area of the larger solid. Part 1: find the similarity ratio by taking the cube root of each volume. Show your work. Part 2: use your answer from part 1 to find the ratio of the surface areas. Show your work. Part 3: set up a proportion and solve to find the surface area of the larger solid.

Answers

Answer:

see below

Step-by-step explanation:

Part 1:

(512) ^ 1/3

-------------------

(2197) ^ 1/3

8

-----

13

The scale factor is 8:13

Part 2

The ratios of the areas is related by scale factor squared

8^2

-----

13^2

64      

------  

169

Part 3

64       960

------  = ----------------

169     SA larger

Using cross products

64 * SA = 169 * 960

64 SA = 162240

Divide each side by 64

64 SA/ 64 = 162240 / 64

SA = 2535

2535 cm^2

Find the missing segment in the image below

Answers

Answer:

8? not sure tho....

Step-by-step explanation:

The answer is 8 trust me on this

PLEASE HELP QUICK 30 POINTS !!!!!!

Ryan wants to make a triangular deck. He wants each side to be a different length.
Select three lengths he could use to make a triangle:
Side 1:
Side 2:
Side 3:
:: 3 feet
:: 10 feet
.: 20 feet
:: 25 feet
:: 60 feet

Answers

Answer:

10 ft, 20ft and 25 ft.

Step-by-step explanation:

10 ft, 20ft and 25 ft.

This will make a triangle as 25 < (10 + 20).

Answer:

Solution given:

one side of a triangle must be less than the sum of two other side .

by making these sense:

3<10+20

10<20+3

20<10+3not true

these three side are not possible.

again

10<20+25

20<25+10

25<20+10

these three side are true so

required side are:

Side 1:10

Side 2:25

Side 3:20

A wind turbine has blades 50m in diameter and an overall height (to the highest point) of 125m. If it has four blades instead of three, create four equations modelling the height of a point on the tip for each of the four blades.

Answers

Answer:

Blade A : H(θ) = 75 + 50 sin θ

Blade B : H(θ) = 75 + 50 sin(θ + 90° )

Blade C : H(θ) = 75 + 50 sin( θ + 180° )  

Blade D : H(θ) = 75 + 50 sin( θ + 270° )

Step-by-step explanation:

Given data :

Diameter of blade = 50 m

overall height = 125 m

The four blades : Blade A , Blade B, Blade C, Blade D  all moves in same direction hence they make 90° to each other.

Lets assume The blades are standing at  θ with the horizontal

The four equation  modelling the heights :

Blade A : H(θ) = 75 + 50 sin θ

Blade B : H(θ) = 75 + 50 sin(θ + 90° )

Blade C : H(θ) = 75 + 50 sin( θ + 180° )  

Blade D : H(θ) = 75 + 50 sin( θ + 270° )

HELP ME WITH THIS PLEASE PLEASE SHOW ME THE FORMULA FOR LETTER C​

Answers

Answer:

Which subject is this . please tell

Answer:

see explanation

Step-by-step explanation:

1

(a)

Calculate slope m using the slope formula

m = [tex]\frac{y_{2}-y_{1} }{x_{2}-x_{1} }[/tex]

with (x₁, y₁  = (8, 3) and (x₂, y₂ ) = (10, 7)

m = [tex]\frac{7-3}{10-8}[/tex] = [tex]\frac{4}{2}[/tex] = 2

(b)

Given a line with slope m then the slope of a line perpendicular to it is

[tex]m_{perpendicular}[/tex] = - [tex]\frac{1}{m}[/tex] = - [tex]\frac{1}{2}[/tex]

(c)

The equation of a line in slope- intercept form is

y = mx + c ( m is the slope and c the y- intercept )

Here m = - [tex]\frac{1}{2}[/tex] , then

y = - [tex]\frac{1}{2}[/tex] x + c ← is the partial equation

To find c substitute (8, 3) into the partial equation

3 = - 4 + c ⇒ c = 3 + 4 = 7

y = - [tex]\frac{1}{2}[/tex] x + 7 ← equation of perpendicular line

--------------------------------------------------------------------------

2

(a)

with (x₁, y₁ ) = (3, 5) and (x₂, y₂ ) = (4, 4)

m = [tex]\frac{4-5}{4-3}[/tex] = [tex]\frac{-1}{1}[/tex] = - 1

(b)

[tex]m_{perpendicular}[/tex] = - [tex]\frac{1}{-1}[/tex] = 1

(c)

y = x + c ← is the partial equation

To find c substitute (3, 5) into the partial equation

5 = 3 + c ⇒ c = 5 - 3 = 2

y = x + 2 ← equation of perpendicular line

what is x in 8 ^ (x - 1 ) = 16 ?

Please help. ​

Answers

Answer:

x=7/3

Step-by-step explanation:

8 ^ (x - 1 ) = 16

We need to rewrite 8 as 2^3  and 16 as 2^4

2^3 ^ (x - 1 ) = 2^4

We know that a^b^c = a^(b*c)

2^(3(x-1)) = 2^4

The bases are the same so the exponents are the same

3(x-1) = 4

Distribute

3x-3 = 4

Add 3 to each side

3x-3+3 = 4+3

3x = 7

Divide by 3

3x/3 = 7/3

x=7/3

What is the area of this triangle

Answers

Answer:

14

Step-by-step explanation:

7*4*1/2=14

Find the expected value of the winnings
from a game that has the following
payout probability distribution:
Payout ($) 0 5
8
10
15
Probability 0.5 0.2 0.15 0.1 0.05
Expected Value = [?]

Answers

Multiply each payout by the corresponding probability and take the total:

E[X] = 0×0.5 + 5×0.2 + 8×0.15 + 10×0.1 + 15×0.05 = 3.95

The expected value = 3.92

What is expected value ?

"It describes the average of a discrete set of variables based on their associated probabilities."

Formula of expected value:

[tex]E(x)=\Sigma[ xP(x)][/tex]

Multiply each value of the random variable by its probability and add the products.

For given question,

We have been given a payout probability distribution.

We need to find the expected value of the winnings.

First we multiply each value of the random variable by its probability .

0 × 0.5 = 0

5 × 0.2 = 1

8 × 0.15 = 1.2

10 × 0.1 = 1

15 × 0.05 = 0.75

Now, we find the sum of above products.

0 + 1 + 1.2 + 1 + 0.75 = 3.95

By using the formula of expected value,

[tex]\Rightarrow E(x)=\Sigma[ xP(x)]\\\\\Rightarrow E(x)=3.92[/tex]

Therefore, the expected value = 3.92

Learn more about the expected value here:

https://brainly.com/question/18523098

#SPJ3

How do you write 50% as a fraction?

Answers

Answer:

[tex]\frac{1}{2}[/tex]

Step-by-step explanation:

Answer:

50% as a fraction would be 1/2

Point T Is the incenter of PQR.Point T is the point of concurrency of the angle bisector. Find ST.

Answers

ST is 10 because since T is the incenter is equidistant from all sides.

Select the true statement about triangle ABC.

A. cos A = cos C
B. cos A = sin C
C. cos A = sin B
D. cos A = tan C

Answers

Answer:

B

Step-by-step explanation:

cosA = [tex]\frac{adjacent}{hypotenuse}[/tex] = [tex]\frac{AB}{AC}[/tex] = [tex]\frac{12}{13}[/tex]

cosC = [tex]\frac{BC}{AC}[/tex] = [tex]\frac{5}{13}[/tex]

sinC = [tex]\frac{opposite}{hypotenuse}[/tex] = [tex]\frac{AB}{AC}[/tex] = [tex]\frac{12}{13}[/tex]

tanC = [tex]\frac{opposite}{adjacent}[/tex] = [tex]\frac{AB}{BC}[/tex] = [tex]\frac{12}{5}[/tex]

Then

cosA = sinC → B

What is the range of f(x) = x - 2, if the domain is {3, 1, 2)?​

Answers

Answer:

Range: {1, -1, 0}

Step-by-step explanation:

Just subtract 2 from each number :)

i need help witht he second part ITS URGENT PLS

Answers

Answer:

7

Step-by-step explanation:

when multiplying powers with the same base, add the powers

Answer:

a²×a⁵

a²⁺⁵

a⁷

OAmalOHopeO

The number of lines of symmetry of an equilateral triangle is _______.

Question 5 options:

A: 4


B: 1


C: 2


D: 3

Answers

Answer:

An equilateral triangle has three lines of symmetr

Step-by-step explanation:

letter d

Answer: 3

Step-by-step explanation: i took test

The width of a rectangle is twice as long as the length. if the length is increased by 50% and the width is decreased by 20%, the perimeter becomes 248. find the width and length of the original rectangle.

Answers

Answer:

Step-by-step explanation:

The percents here make this more tricky than it originally seems to be. We'll make a table and see where it takes us:

                          original                 new

length

width

And we'll fill it in according to our rules given. Starting with the original, we are told that the width is twice as long as the length. We don't know the length, so we'll call that L, and if the width is twice that, the width is 2L:

                           original                  new

length                       L

width                       2L

Now here's the tricky part. What I'm going to do is fill in the "new" column with the expressions and then we'll simplify them in the next step.

The length is increased by 50%. So we have 100% of the original length and we are adding another 50% to that:

                            original                        new

length                      L                       100%L + 50%L

width                       2L

The width is decreased by 20%, so we have 100% of 2L and we are subtracting 20% of 2L from that:

                             original                       new

length                        L                     100%L + 50%L

width                        2L                  100%(2L) - 20%(2L)

And now we'll simplify that "new" column:

                              original                         new

length                        L                         150%L = 1.5L

width                         2L               80%(2L) = 160%L = 1.6L

Now we're ready for the perimeter part. The formula for the perimeter of a rectangle is P = 2L + 2w, so filling in from our "new" column, since 248 is the perimeter given for AFTER the rectangle's length and width are manipulated:

248 = 2(1.5L) + 2(1.6L) and

248 = 3L + 3.2L and

248 = 6.2L so

L = 40 and that means that w = 80 (because in the "original" column, the width is twice the length)

What is 1/4 0.75 1/3 0.5 greatest to least

Answers

Answer:

1/4 = 1 ÷ 4 = 0.251/3 = 1 ÷ 3 ≈ 0.330.750.5

0.75 → 0.5 → 1/3(0.33) → 1/4(0.25)

hope this helps! feel free to clarify

Need help please need it quick

Answers

Answer:

Choice A

Step-by-step explanation:

An arithmetic sequence is one which has an incremental change.

for our answer, our change is +5

option a is the correct answer

3. A rectangle has a length of 2x – 9 and a width of x2 + 3x – 4. What is the polynomial that models the area
of the rectangle?

Answers

Answer:

(C) 2x^3 - 3x^2 - 35x + 36

Step-by-step explanation:

First multiply 2x by x^2 + 3x - 4:

(2x)(x^2 + 3x - 4)

2x^3 + 6x^2 - 8x

Next multiply -9 by x^2 + 3x - 4:

(-9)(x^2 + 3x - 4)

-9x^2 - 27x +36

Now add the two polynomials by adding like terms:

(2x^3 + 6x^2 - 8x) + (-9x^2 - 27x +36)

2x^3 + 6x^2 - 9x^2 - 8x - 27x + 36

2x^3 - 3x^2 - 35x + 36

Hope this helps (●'◡'●)

pls find the value of X ​

Answers

Answer: 64°

Step-by-step explanation:

Since QS = RS, ∠Q = ∠R = 32°∠QSR = 180° - 32° - 32° = 116°∠QSR = ∠NSP = 116°Since MN║PS, ∠NSP + ∠N = 180° ∠N = 180° - 116° = 64°

Answer:

x = 64°

Step-by-step explanation:

Since the triangle is isosceles, the base angles are congruent, that is

∠ R = ∠ Q = 32° , then

vertex angle = 180° - (32 + 32)° = 180° - 64° = 116°

∠ NSR = 116° ( vertically opposite angle )

∠ MNS and ∠ NSR are same- side interior angles and sum to 180° , then

x = 180° - 116° = 64°

Help please…..?!!!!!☺️

Answers

Answer:

Step-by-step explanation:

Sample space of die 1 = {1,2,2,3,3,4}

Sample space of die 2 = {1,3,4,5,6,8}.

The highest value that we get by rolling die 1 is 4 and die 2 is 8, which give the sum 12. So, we cannot have any other combinations other than this , as the highest value itself gives 12.

Rolling a sum of 12 with the dice = {(4,8) }

Probability = 1/36

Help me to prove it

Answers

Answer:

see explanation

Step-by-step explanation:

Using the identities

cotA = [tex]\frac{1}{tanA}[/tex]

cot²A = cosec²A - 1

tan²A = sec²A - 1

Consider the left side

(cotA + tanA)² ← expand using FOIL

= cot²A + 2cotAtanA + tan²A

= cosec²A - 1 + 2 .[tex]\frac{1}{tanA}[/tex] . tanA + sec²A - 1

= cosec²A - 1 + 2 + sec²A - 1

= sec²A + cosec²A - 2 + 2

= sec²A + cosec²A

= right side, thus proven

Options:
Rotation
Reflection
Translation

Answers

Answer: Reflection

Step-by-step explanation: Please mark brainliest

if a=(p+q),b=(p-q)and c=qsquare -psquare, show that ab+c=0​

Answers

Answer:

I think this is the ans

Step-by-step explanation:

ab+c=0

(p+q)(p-q)=0

p Square-q Square =0

0=p Square-q Square

Other Questions
Select the correct answer. What is the completely factored form of this polynomial? 5x4 + 20x3 105x2 A. 5x3(x + 3)(x 7) B. 5x2(x 3)(x + 7) C. 5x3(x 3)(x + 7) D. 5x2(x + 3)(x 7) the balance of payment between Vietnam and Iran Un ingeniero civil a cargo de una obra contrata una cierta cantidad de obreros. Estima que, si lepagara S/ 1.400 a cada uno de los obreros, le faltaran S/ 8.000, pero si le pagara S/ 1.200 a cadauno, le sobraran S/ 2.000. El nmero de obreros que contrat es Find the income tax of the following cases annual salary=8000,allowance=1450,rate of tax=12 Helpppppppp pleaseeee What type of energy is associated with this movement of the molecules? time in months it would take for a $1500 dollar investment in a TFSA to grow to $1545 if the simple interest at a rate paid was 2% per annum. A company pays $20 per hour for up to 8 hours of work, and $30 per hour for overtime hours (hours beyond 8 hours). For up to 8 hours worked, the equation for total pay (y) for hours worked (x) is y = 20x. For over 8 hours worked, what is the equation for total pay (y) as a function of total hours worked (x)? Solve for x. Type your answer below.5(2x12)=90 Given the following coordinates complete the glide reflection transformation. Alain is a sales representative for an established awnings manufacturer. Business is good, but he is concerned that the company has spent little on new product development and has not created a new product in over five years. Without new products, Alain can market his current products only to his current customers or Multiple Choice diversify. intensify his prototyping. expand his early adopter market segment. market the same products to similar customers. given m||n, find the value of x For the right angle, find the missing quantity indicated below the figure. First Conditional : "I will go if they go." Second conditional : He said he _____ if they ______ Which phrase describes the linear relationship between the values of x and y shown in the table? x l y -3 l 90 l 0 3 l -9 A. y is 3 times x B. x is 6 more than y C. y is 12 more than x D. y is -3 times x Help me plz its due today What are some long-term effects in the slaughterhouse cases The number of revolutions made by a bicycle wheel of 28-inch diameter in travelling 1/2 mile is closest to:(A) 720(B) 180C) 360D) 120 An error due to wrong input by the user Which of the following statements about the monetary aggregates is true? a. The growth rates of M1 and M2 always track each other closely. b. M1 is greater than M2. c. When the growth rate of M2 increases, the growth rate of M1 must also increase. d. When you transfer funds from your savings account to your checking account, M1 increases and M2 stays the same.Given that, in billions of U.S. dollars, we have in currency, in demand deposits, in traveler's checks, in savings deposits, and in other checkable deposits. The total M1 amount in this economy is $__________